Questions from Prep Test 29 (Oct 1999) Forum

Prepare for the LSAT or discuss it with others in this forum.
Post Reply
lavenderb181

New
Posts: 16
Joined: Sat Apr 30, 2011 2:59 pm

Questions from Prep Test 29 (Oct 1999)

Post by lavenderb181 » Sat Apr 30, 2011 3:58 pm

Hey all-

I am frantically studying for the June LSAT, hoping that I can be prepared in time. This forum has proved to be a really useful resource (when I'm not wasting time on it, that is). I have a few questions from this prac test ... who wants to help me out?! :)

Section 1 LR #16. We can learn about the living conditions of a vanished culture by examining its language. Thus, it is likely that the people who spoke Proto-Indo-European, the language from which all Indo-European languages descended, lived in a cold climate, isolated from ocean or sea, because Proto-Indo-European lacks a word for "sea," yet contains words for "winter," "snow," and "wolf."

Which one of the following, if true, most seriously weakens the argument?
A) A word meaning "fish" was used by the people who spoke Proto-Indo-European.
B) Some languages lack words for prominent elements of the environments of their speakers.
D) Proto-Indo-European possesses words for "heat."

Correct answer is B, which I selected, but I couldn't quite figure out why A and D were wrong. I mainly eliminated A and D because they are different versions of the same answer, so if one was right then the other would have to be too. Why is B the best answer though - is it because it is the most all-encompassing answer?


Last logic game (#20-24)


I'm having trouble making the inferences on this game, and thus cannot confidently answer any of the questions, except the first one. Could somebody point out the inferences or explain the best way to set up this game?

Exactly six piano classes are given sequentially on Monday: two with more than one student and four with exactly one student. Exactly four females - G, H, I, and K, - and five males - L, N, O, P, and S - attend these classes. Each students attends exactly one class. The following must obtain:
I and L together constitute one class.
P and exactly two other together constitute one class.
K is the first female, but not the first student, to attend a class.
G's class is at some time after I's but at some time before P's.
O's class is at some time after G's.


Section 4 LR #11


Sometimes when their trainer gives the hand signal for "Do something creative together," two dolphins circle a pool in tandem and then leap through the air simultaneously. On other occasions the same signal elicits synchronized backward swims or tail-waving. These behaviors are not simply learned responses to a given stimulus. Rather, dolphins are capable of higher cognitive functions that may include the use of language and forethought.

Which one of the following, if true, most strengthens the argument?
B) The dolphins often exhibit complex new responses to the hand signal. (Correct answer)
D) Dolphins do not interact with humans the way they interact with one another.

None of the answer choices really seemed correct. Could somebody explain how B strengthens the argument?


Thank you!

User avatar
JamMasterJ

Platinum
Posts: 6649
Joined: Sat Jan 29, 2011 7:17 pm

Re: Questions from Prep Test 29 (Oct 1999)

Post by JamMasterJ » Sat Apr 30, 2011 4:29 pm

lavenderb181 wrote: Section 1 LR #16. We can learn about the living conditions of a vanished culture by examining its language. Thus, it is likely that the people who spoke Proto-Indo-European, the language from which all Indo-European languages descended, lived in a cold climate, isolated from ocean or sea, because Proto-Indo-European lacks a word for "sea," yet contains words for "winter," "snow," and "wolf."

Which one of the following, if true, most seriously weakens the argument?
A) A word meaning "fish" was used by the people who spoke Proto-Indo-European.
B) Some languages lack words for prominent elements of the environments of their speakers.
D) Proto-Indo-European possesses words for "heat."
The reason that a is out is because fish can live in lakes and streams. This is the same reason that d is out; you cany get heat from other sources than the temperature, i.e. fire. The point is that your looking for something that weakens the response, and there is a plausible logical argument against either of those being right. B is TCR because it doesn't have loopholes; it is a principle that opens a possibilty for a flaw in the stimulus. It doesn't prove that the stimulus is incorrect, but it makes it possible that the stimulus could be incorrect.

User avatar
JamMasterJ

Platinum
Posts: 6649
Joined: Sat Jan 29, 2011 7:17 pm

Re: Questions from Prep Test 29 (Oct 1999)

Post by JamMasterJ » Sat Apr 30, 2011 4:36 pm

lavenderb181 wrote: Section 4 LR #11

Sometimes when their trainer gives the hand signal for "Do something creative together," two dolphins circle a pool in tandem and then leap through the air simultaneously. On other occasions the same signal elicits synchronized backward swims or tail-waving. These behaviors are not simply learned responses to a given stimulus. Rather, dolphins are capable of higher cognitive functions that may include the use of language and forethought.

Which one of the following, if true, most strengthens the argument?
B) The dolphins often exhibit complex new responses to the hand signal. (Correct answer)
D) Dolphins do not interact with humans the way they interact with one another.
Thank you!
If the Dolphins were circling the pool and leaping everytime the "do something creative" signal was given, it would be akin to a dog rolling over every time that a specific signal was given (not a higher cognitive function, just a stimulus response). Because they are doing something new, it constitutes a choice made by the dolphins. I think that there is still a weakness in the argument, that there could be an alternate source of these complex new responses than actually possessing higher cognitive functions. However, in strengthen questions, the argument must only be strengthened to a miniscule degree, which this answer does.

User avatar
JamMasterJ

Platinum
Posts: 6649
Joined: Sat Jan 29, 2011 7:17 pm

Re: Questions from Prep Test 29 (Oct 1999)

Post by JamMasterJ » Sat Apr 30, 2011 5:44 pm

\
lavenderb181 wrote: Last logic game (#20-24)
Exactly six piano classes are given sequentially on Monday: two with more than one student and four with exactly one student. Exactly four females - G, H, I, and K, - and five males - L, N, O, P, and S - attend these classes. Each students attends exactly one class. The following must obtain:
I and L together constitute one class. P and exactly two other together constitute one class.
K is the first female, but not the first student, to attend a class. G's class is at some time after I's but at some time before P's. O's class is at some time after G's.
Inferences:
I > G > P. Also G > O
K is not first and no other girl is first or second
K rule means neither is first or second
Of the classes with multiple students, IL is one and P is in the other with two students other than K, G, I, L. This can be N, S, H, and O, but not N and S, b/c one of them must be first.
1: Not K. Therefore, not, G, H, IL, K. Because of rules precluding girls, O, and P, N or S must be here
2: Not G, H, IL
3: Not G. This is the earliest that IL can be. K must be either 3 or 2.
4:IL must be either 3 or 4. This is the earliest G can be.
5:
6:

The biggest things in this game are figuring out where to put K and the IL block. You can do this by making three hypos, one with K=2 and IL =3, one with K=2 and IL = 4, and one with K=3 and IL=4. If IL is 4, then G is 5 and O and P are 6

It was hard to see without having the actual question in front of me so if I screwed up, I'm sorry

lavenderb181

New
Posts: 16
Joined: Sat Apr 30, 2011 2:59 pm

Re: Questions from Prep Test 29 (Oct 1999)

Post by lavenderb181 » Sun May 01, 2011 4:16 pm

Thank you!

Another question ---

from page 153 in the Logic Games Bible (from Oct 2004 LSAT, #13-17).
One of the game's conditions is: "The site visited third dates from a more recent century than does either the site visited first or that visited fourth."

In the answer explanations, it says you can infer that the site visited third dates from a more recent century than BOTH the sites visited first and fourth. I know that on the LSAT "either/or" can mean either, or , or both, but it doesn't have to, right?

Here's the part from the explanation, in case I mis-interpreted it...
"The last rule states that the third sites dates from a more recent century than either the first or fourth site. Thus, the third site cannot date from the 8th century, and the first and fourth sites cannot date from the 10th century." [options are 8th, 9th and 10th century for this game].

Thanks!

User avatar
zdamico

Bronze
Posts: 165
Joined: Mon Aug 01, 2011 7:57 pm

Re: Questions from Prep Test 29 (Oct 1999)

Post by zdamico » Thu Aug 04, 2011 3:51 pm

I have a question from this section and figured it would be better to post it in here than starting a new topic.

Number 22 on Section #4, The LR. I put B and the correct answer is C. I am probably just missing something obvious, and am prepared to feel stupid, but can someone explain why this is?

Edit: I was in a rush and missed the CANNOT. Damn. Dropped me a point. :(

Want to continue reading?

Register now to search topics and post comments!

Absolutely FREE!


Post Reply

Return to “LSAT Prep and Discussion Forum”